1answer.
Ask question
Login Signup
Ask question
All categories
  • English
  • Mathematics
  • Social Studies
  • Business
  • History
  • Health
  • Geography
  • Biology
  • Physics
  • Chemistry
  • Computers and Technology
  • Arts
  • World Languages
  • Spanish
  • French
  • German
  • Advanced Placement (AP)
  • SAT
  • Medicine
  • Law
  • Engineering
Amiraneli [1.4K]
3 years ago
5

PLEASE HELP!! WILL GIVE CROWN. ZOOM INTO PICTURE IF NEED TO SEE​

Mathematics
2 answers:
Korolek [52]3 years ago
7 0

Answer:

k= -4

Step-by-step explanation:

Finger [1]3 years ago
7 0

k = 4

adding 4 to f(x) will make the line shift up 4 points on the y-axis, therefore making the equation equal g(x)

You might be interested in
What are the solutions of the compound inequality -6<=2x-4<4? Graph the solutions
Nata [24]

-6\leq2x-4 < 4\qquad\text{add 4 to both sides}\\\\-2\leq2x < 8\qquad\text{divide both sides by 2}\\\\-1\leq x < 4

5 0
3 years ago
I'm a little stupid towards math any helpers?​
Allisa [31]

Answer:

I beleive it is the first one, but I am not 100% sure. Sorry

Step-by-step explanation:

5 0
3 years ago
Hello, Brainly community!
ioda

Answer:

(B)  \displaystyle \frac{W(3.1) - W(2.9)}{0.2}

General Formulas and Concepts:

<u>Calculus</u>

Limits

Derivatives

  • The definition of a derivative is the slope of the tangent line.

Derivative Notation

Instantaneous Rates

  • Tangent Line: \displaystyle f'(x) = \frac{f(b) - f(a)}{b - a}

Step-by-step explanation:

Since we are trying to find a <em>rate</em> at which W(t) changes, we must find the <em>derivative</em> at <em>t</em> = 3.

We are given 2 close answer choices that would have the same <em>numerical</em> answer but different <em>meanings</em>:

  1. (A)  \displaystyle  \lim_{t \to 3} W(t)
  2. (B)  \displaystyle \frac{W(3.1) - W(2.9)}{0.2}

If we look at answer choice (A), we see that our units would simply just be volume. It would not have the units of a rate of change. Yes, it may be the closest numerically correct answer, but it does not tell us the <em>rate</em> at which the volume would be changing and it is not a derivative.

If we look at answer choice (B), we see that our units would be cm³/s, and that is most certainly a rate of change. Answer choice (B) is also a <em>derivative</em> at <em>t</em> = 3, and a derivative tells us what <em>rate</em> something is changing.

∴ Answer choice (B) will give us the best estimate for the value of the instantaneous rate of change of W(t) when <em>t</em> = 3.

Topic: AP Calculus AB/BC (Calculus I/I + II)

Unit: Differentiation

Book: College Calculus 10e

8 0
3 years ago
3(x-4)=12x distributing or diving
frutty [35]
Distribution; 3x-12=12x
7 0
3 years ago
Guys, Can you please help me with these questions
MakcuM [25]

Answer:

a) w^{13} x^{5} y^{6}

b) \frac{x}{3y^{6} }

Step-by-step explanation:

a) (w^{2} xy^{3} )^{2}(w^{3}x )^{3}

1. Distribute the second power (2) outside the first pair of parenthesis:

(w^{2(2)} x^{2} y^{3(2)} )

= w^{4} x^{2} y^{6} (w^{3}x )^{3}

2. Distribute the third power (3) outside the second pair of parenthesis:

(w^{3(3)} x^{3} )

= w^{4} x^{2} y^{6} w^{9} x^{3}

3. Combine like terms:

w^{13} x^{5} y^{6}

--------------------------------------------

b) \frac{2x^{2} y^{5} }{6xy^{11} }

1. Factor the number 6 (= 2 · 3):

\frac{2x^{2} y^{5} }{2(3)xy^{11} }

2. Cancel the common factor (2):

\frac{x^{2} y^{5} }{3xy^{11} }

3. Cancel out xy^{5} in the numerator an denominator:

\frac{x}{3y^{6} }

hope this helps!

4 0
2 years ago
Other questions:
  • The accompanying data are the times to failure (in millions per cycle) of high-speed turbine engine bearings made out of two dif
    12·1 answer
  • Ed and Sally are taking a tour of Paris. There are 3 different types of tours (walking tour, bike tour, and trolley tour) and 2
    10·2 answers
  • Simplify square root of 5 open parentheses 10 minus 4 square root of 2 close parentheses
    12·1 answer
  • How do you do long division
    10·1 answer
  • Fourteen pounds of beans are distributed equally into 10 bags to give out at the food bank. How many pounds of beans are in each
    12·1 answer
  • One sixpack of soda has six cans. How many cans are there in six sixpacks of soda?
    11·2 answers
  • Plz help me as soon as possible! Thank you
    7·2 answers
  • Help as soon as possible!!<br><br> Answer only if you know! <br><br> At least answer one of them!
    6·1 answer
  • Offering brainliest!!! Find the Volume of each cylinder pictured below. Use 3.14 for π and round to the nearest tenth:
    10·1 answer
  • To prove that the triangles are similar by the SAS similarity theorem, it needs to be shown that
    5·1 answer
Add answer
Login
Not registered? Fast signup
Signup
Login Signup
Ask question!